LSAT and Law School Admissions Forum

Get expert LSAT preparation and law school admissions advice from PowerScore Test Preparation.

 Administrator
PowerScore Staff
  • PowerScore Staff
  • Posts: 8929
  • Joined: Feb 02, 2011
|
#41235
Complete Question Explanation
(The complete setup for this game can be found here: lsat/viewtopic.php?t=4995)

The correct answer choice is (A)

If J and K lead morning sessions, then the morning/afternoon assignment of the sessions is established:
PT66_J12_Game_#1_#4_diagram 1.png
The linear component of the game is still dictated by the JL :longline: NO relationship, but the placement of the blocks to the days is not fully set. However, this information is not necessary to answer the question, because answer choice (A) attempts to have L’s session meet in the morning, which is impossible since L must be paired with J, and J must lead a morning session per the question stem. Consequently, answer choice (A) cannot occur, and must be the correct answer.
You do not have the required permissions to view the files attached to this post.
 cnoury1221
  • Posts: 15
  • Joined: Jun 24, 2019
|
#67160
Hello,
This question caught me off guard as I noticed J leads 2 sessions, and so R leads none. My mistake was thinking that all the lab assistants have to lead at least one session. How can I decipher this in the language of the stimulus?

Thanks!
Carolyn
 James Finch
PowerScore Staff
  • PowerScore Staff
  • Posts: 943
  • Joined: Sep 06, 2017
|
#67193
Hi CNoury,

Right off the bat, I see you made a mistake: the setup states that each session will be lead by a different assistant, and then gives us exactly as many assistants as sessions, meaning that each assistant will lead exactly one session. This means that K and R make up one day, O and N be on one of the last two days (as O can't be on the same day as L or J) and the other day will be J and L. With that in mind, this particular question gives us daily blocks of (morning first, afternoon second):

K--R,

J--L,

O--N

With the only other consideration that the J block must come before the O block (KR can go anywhere). Based on that diagram, and looking through the answer choices as what cannot be true, answer choice (A) immediately presents us with an issue: L must always go with J, so if J is leading a morning session L can't. So L cannot go on Wednesday morning, making (A) correct.

Hope this clears things up!
 cnoury1221
  • Posts: 15
  • Joined: Jun 24, 2019
|
#67520
James Finch wrote:Hi CNoury,

Right off the bat, I see you made a mistake: the setup states that each session will be lead by a different assistant, and then gives us exactly as many assistants as sessions, meaning that each assistant will lead exactly one session. This means that K and R make up one day, J and L/N make up another, and O and L/N make up the last day. With that in mind, this particular question gives us daily blocks of (morning first, afternoon second):

K--R,

J--N/L,

O/L--O/L/N

With the only other consideration that the J block must come before the O block (KR can go anywhere). Based on that diagram, and looking through the answer choices as what cannot be true, answer choice (A) immediately presents us with an issue: L can only go in the morning when paired with O, which in turn can only go on Thursday or Friday. So L cannot go on Wednesday morning, making (A) correct.

Hope this clears things up!

Hi James,
Thank you for the help!! I have come back to review this game and for some reason this question got me again:
My setup to finally get (A) was the following ( morning first and afternoon second):

K -- R ( Wednesday)
J -- L ( Thursday)
O -- N ( Friday)

J -- L ( Wednesday)
O--N ( Thursday)
K -- R (Friday)

With these diagrams I was able to see that (A) , L's session on Wednesday morning cannot happen with J and K holding morning sessions.

Are these diagrams ok?

Thank You!

Carolyn
 James Finch
PowerScore Staff
  • PowerScore Staff
  • Posts: 943
  • Joined: Sep 06, 2017
|
#67546
Hi Carolyn,

I actually missed one of the rules when I explained it above, and edited my answer to you; the pairs are actually set, as L and O can't go together, meaning L and J must always be together and O and N must as well. So there will always be a O--N block on Thursday or Friday; if Thursday, then the L/J block will go on Wednesday and the K/R block on Friday. But if we push the O--N block to Friday, the L/J and K/R can switch position in Wednesday and Thursday.

As to this specific question, your diagrams are correct but leave out the possibility of K/R on Wednesday, or:

J -- L ( Wednesday)
K -- R ( Thursday)
O--N (Friday)

Hope this helps!

Get the most out of your LSAT Prep Plus subscription.

Analyze and track your performance with our Testing and Analytics Package.